De Broglie wavelength - relativistic correction

In summary, the de Broglie wave equation states that the wavelength is related to the momentum by a constant, h.
  • #1
astrof
16
0
According to the wiki page:
http://en.wikipedia.org/wiki/Matter_wave
the generalised form for the de Broglie wave is simply:
[tex] \lambda = h/p[/tex]

I suppose this not correct, because there is no trasform which can change only one side of the equation.
In this case we have two variables:
1. a momentum: [tex]p = mv[/tex]
and
2. a wavelength: [tex]\lambda[/tex]

the h is just a number - constant.

Therefore the correct, transformed version, of the whole equation is:
[tex]\gamma\lambda = h/{\gamma mv}[/tex]

thus the final - general the de Broglie relation is:
[tex]\gamma^2\lambda = h/{mv}[/tex]
or:
[tex]mv\lambda = h(1-v^2/c^2)[/tex]

So, what you think about my discovery, proposition? :)

Is there possible to resolve (experimentally) which form is correct?
 
Physics news on Phys.org
  • #2
The usual formula for the DeBroglie wavelength is correct

[itex]p=h/\lambda[/itex].

The expression for momentum changes from [itex]mv[/itex] to [itex]\gamma mv[/itex], but the relationship between momentum and wavelength remains the same.

In fact, the DeBroglie formula was derived with special relativity.

If you have a stationary wavepacket of mass M oscillating at frequency [itex]f[/itex], such that its energy [itex]E[/itex] is [itex]h f[/itex], then in a different reference frame, moving with respect to the wave packet, the Lorentz transformation, (and the relativity of simultaneity) shows that that wavepacket will have a momentum [itex]p[/itex] equal to [itex]h[/itex], divided by the observed wavelength [itex]\lambda[/itex].
It's one of the more elegant findings in the early days of quantum physics.
 
  • #3
Indeed. But this transformed version of the equation is mathemathicaly inconsistent.

Therefore my claym is: the wavelength should be transformed too.

Disprove my claim.

BTW. The radiation of accelerated charge is proportional to the gamma^2 also.

And the radiation is just prominent example of the wave phenomena.
So, now I have a very strong argument!
 
Last edited:
  • #4
Don't bother with old theories outdated since 1925! Learn quantum mechanics!
 
  • #5
astrof said:
Therefore the correct, transformed version, of the whole equation is:
[tex]\gamma\lambda = h/{\gamma mv}[/tex]
How did you come up with the left-hand side of that equation?
 
  • #6
astrof said:
According to the wiki page:
http://en.wikipedia.org/wiki/Matter_wave
the generalised form for the de Broglie wave is simply:
[tex] \lambda = h/p[/tex]

I suppose this not correct, because there is no transform which can change only one side of the equation.

I'm not sure what inconsistency you're hinting about, but let me sketch how something like the de Broglie relation is true relativistically.

Instead of using wavelength, it's more convenient to deal with wave number [itex]k[/itex], which in one dimension is related to wavelength through:

[itex]k = \frac{2\pi}{\lambda}[/itex]

In terms of [itex]k[/itex], the de Broglie relation is just:

[itex]k = frac{p}{\hbar}[/itex]

where [itex]\hbar = \frac{h}{2\pi}[/itex]

Under a Lorentz transformation, it isn't simply that [itex]p \Rightarrow \gamma p[/itex]. The general relationship is (in one spatial dimension):

[itex]p' = \gamma (p - \frac{v}{c^2} E)[/itex]
[itex]E' = \gamma (E - v p)[/itex]

[itex]k[/itex] and [itex]\omega[/itex] transform under Lorentz transforms similarly:

[itex]k' = \gamma (k - \frac{v}{c^2} \omega)[/itex]
[itex]\omega' = \gamma(\omega - k v)[/itex]

What's invariant under Lorentz transformations is the phase of the wave. The phase is given by:

[itex]\phi = k x - \omega t[/itex]

The pair [itex](k,\omega)[/itex] and [itex](x,t)[/itex] both transform under Lorentz transformations in a way that makes the phase invariant.
 
  • #7
Nugatory said:
How did you come up with the left-hand side of that equation?
I simpy transforemed not the momentum only, but the length too, using the same transformation: the Lorentz's transform.

I remember an obvious fact:
a length - distance is not invariant under the Lorentz transformation!

Maybe I show some further investigation.
The standard formula for a circular motion is:
[tex]pv = k/r[/tex]
and the common de Broglie condition:
[tex]pr = n h/2\pi = n \hbar[/tex]

so, from the first equation: r = k/pv, thus the second eq.:
[tex]pk/pv = k/v = n\hbar \to\ v = k/n\hbar = c\alpha/n \approx c/137n[/tex]

And there is just nothing new: the relativistic corrections disappears completely!
But that is impossible, because we know already the fast - relativistic
particles behave in other way... due to the relativistic effects,
therefore these effects can't and don't cancel so silently!
 
Last edited:
  • #8
stevendaryl said:
The pair [itex](k,\omega)[/itex] and [itex](x,t)[/itex] both transform under Lorentz transformations in a way that makes the phase invariant.

That is nothing, because the phase is invariant under the Galilean transformation too!
 
  • #9
The wave equation is not Galilei invariant. Check it!
 
  • #10
vanhees71 said:
The wave equation is not Galilei invariant. Check it!

I told a phase is invariant, and under any transformation, therefore it's irrelevant in the context of the relativistic effects.
But a distance is not invariant under Lorentz, thus the wavelength, in the de Broglie relation,
which is a longitudinal distance, should be transformed too; not the momentum alone.
 
  • #11
astrof said:
I told a phase is invariant, and under any transformation, therefore it's irrelevant in the context of the relativistic effects.

It's relevant to the extent that it shows you are wrong: Both [itex]p[/itex] and [itex]k = \frac{2\pi}{\lambda}[/itex] transform under Lorentz transformations in exactly the same way, so it continues to be true that [itex]p = \hbar k = \frac{2 \pi \hbar}{\lambda}[/itex]
 
  • #12
astrof said:
That is nothing, because the phase is invariant under the Galilean transformation too!

So what? The point is that [itex]p = \hbar k = \frac{2 \pi \hbar}{\lambda}[/itex]. [itex]E = \hbar \omega = \frac{2 \pi \hbar}{T}[/itex]. Those equations are always true. Both [itex]p[/itex] and [itex]k[/itex] transform under a Lorentz transformation in exactly the same way:

[itex]p' = \gamma (p - \frac{E}{c^2} v)[/itex]
[itex]E' = \gamma (E - pv)[/itex]

[itex]k' = \gamma (k - \frac{\omega}{c^2})[/itex]
[itex]\omega' = \gamma (\omega - k v)[/itex]

So [itex]p = \hbar k[/itex] and [itex]E = \hbar \omega[/itex] are preserved under Lorentz transformations; if they are true in one frame, they are true in every frame.
 
  • Like
Likes vanhees71
  • #13
astrof said:
I simply transformed not the momentum only, but the length too, using the same transformation: the Lorentz's transform.

No, you didn't. The Lorentz transforms convert coordinates from one frame to another, and can be used to derive the correct formulas for transforming wavelengths and momenta from one frame to another. However, you haven't used these formulas. On the right-hand side you're assuming that ##p'=\gamma{p}## but that's not the correct transformation for momentum; and on the left-hand side you've just multiplied the wavelength by ##\gamma## and that's not the correct transformation for wavelengths.

Take another look at @stevendaryl's analysis, and also pay close attention to the relativity of simultaneity, as @jfizzix suggests.
 
  • #14
stevendaryl said:
It's relevant to the extent that it shows you are wrong: Both [itex]p[/itex] and [itex]k = \frac{2\pi}{\lambda}[/itex] transform under Lorentz transformations in exactly the same way, so it continues to be true that [itex]p = \hbar k = \frac{2 \pi \hbar}{\lambda}[/itex]

You just promote my idea!
Namely: momentum depends explicitly on the wavelength,
therefore the wavelength must be transformed!

If we leave the wavelength intact, then the momentum can't change too!
 
  • #15
astrof said:
You just promote my idea!
Namely: momentum depends explicitly on the wavelength,
therefore the wavelength must be transformed!

If we leave the wavelength intact, then the momentum can't change too!

Yes, but wavelength also changes under Lorentz transformations.
 
  • #16
Nugatory said:
No, you didn't. The Lorentz transforms convert coordinates from one frame to another, and can be used to derive the correct formulas for transforming wavelengths and momenta from one frame to another. However, you haven't used these formulas. On the right-hand side you're assuming that ##p'=\gamma{p}## but that's not the correct transformation for momentum; and on the left-hand side you've just multiplied the wavelength by ##\gamma## and that's not the correct transformation for wavelengths.

I don't care if the transformation is correct or not - it's just a standard form/transform of the momentum in the relativity.
http://en.wikipedia.org/wiki/Energy–momentum_relation
 
Last edited:
  • #17
stevendaryl said:
Yes, but wavelength also changes under Lorentz transformations.

Indeed. Because that fact I transformed the wavelength, in the de Broglie relation, adequately. :)
 
  • #18
astrof said:
I don't care if the transformation is correct or not - it's just a standard form/transform of the momentum in the relativity.
http://en.wikipedia.org/wiki/Energy–momentum_relation
That's silly. If you use an incorrect formula of course you'll get incorrect results, and that's what happened here. The wikipedia article does not support the transform you're using; you have to transform the velocity as well multiplying by ##\gamma##: ##p'=\gamma{m_0}v'## not ##p'=\gamma{m}_0v=\gamma{m}_0{p}##.

We can give you the correct analysis, and you are free not to like it, but the forum rules don't allow continued argument after a mistaken premise has been pointed out. This thread is closed.
 

What is De Broglie wavelength?

The De Broglie wavelength is a concept in quantum mechanics that describes the wavelength of matter particles, such as electrons and protons.

How is De Broglie wavelength calculated?

The De Broglie wavelength is calculated using the following formula: λ = h/mv, where λ is the De Broglie wavelength, h is Planck's constant, m is the mass of the particle, and v is its velocity.

What is the significance of the De Broglie wavelength?

The De Broglie wavelength helps us understand the wave-particle duality of matter. It also allows us to make predictions about the behavior of particles at the quantum level.

What is the relativistic correction for De Broglie wavelength?

The relativistic correction for De Broglie wavelength accounts for the effects of special relativity, such as time dilation and length contraction, on the motion of particles with high velocities.

How is the relativistic correction for De Broglie wavelength calculated?

The relativistic correction for De Broglie wavelength is calculated using the following formula: λ' = λ/(1-(v/c)^2)^1/2, where λ' is the corrected De Broglie wavelength, λ is the original De Broglie wavelength, v is the velocity of the particle, and c is the speed of light.

Similar threads

  • Quantum Interpretations and Foundations
Replies
3
Views
850
  • Quantum Interpretations and Foundations
Replies
6
Views
1K
  • Quantum Interpretations and Foundations
Replies
1
Views
1K
  • Quantum Interpretations and Foundations
Replies
6
Views
2K
  • Quantum Interpretations and Foundations
Replies
28
Views
4K
  • Quantum Interpretations and Foundations
Replies
8
Views
1K
  • Quantum Interpretations and Foundations
Replies
14
Views
2K
  • Other Physics Topics
Replies
5
Views
861
  • Quantum Interpretations and Foundations
Replies
12
Views
1K
  • Quantum Interpretations and Foundations
Replies
3
Views
978
Back
Top